Dejemos que $R$ sea un anillo, donde $a^{3} = a$ para todos $a\in R$ . Demostrar que $R$ debe ser un anillo conmutativo.
Por favor, guíenme con una prueba. Gracias por su amabilidad.
Dejemos que $R$ sea un anillo, donde $a^{3} = a$ para todos $a\in R$ . Demostrar que $R$ debe ser un anillo conmutativo.
Por favor, guíenme con una prueba. Gracias por su amabilidad.
Para empezar
$$ 2x=(2x)^3 =8 x^3=8x \ . $$
Por lo tanto, $6x=0 \ \ \forall x$ .
También
$$ (x+y)=(x+y)^3=x^3+x^2 y + xyx +y x^2 + x y^2 +yxy+ y^2 x + y^3 $$ y
$$ (x-y)=(x-y)^3=x^3-x^2 y - xyx -y x^2 + x y^2 +yxy+ y^2 x -y^3 $$
Restando obtenemos
$$ 2(x^2 y +xyx+yx^2)=0 $$
Multiplica la última relación por $x$ a la izquierda y a la derecha para obtener
$$ 2(xy+x^2yx+xyx^2)=0 \qquad 2(x^2yx+xyx^2+yx)=0 \ . $$
Restando las dos últimas relaciones tenemos
$$ 2(xy-yx)=0 \ . $$
A continuación, demostramos que $3( x+x^2)=0 \ \ \forall x$ . Se obtiene esto de
$$ x+x^2=(x+x^2)^3=x^3+3 x^4+3 x^5+x^6=4(x+x^2) \ . $$
En particular
$$ 3 (x+y +(x+y)^2) =3( x+x^2+ y+ y^2+ xy+yx)=0 \, $$
terminamos con $3(xy+yx)=0$ . Pero como $6xy=0$ tenemos $3(xy-yx)=0$ . A continuación, reste $2(xy-yx)=0$ para conseguir $xy-yx=0$ .
$\rm(1)\quad ab=0\: \Rightarrow\: ba = 0\ \ via\ \ ba = (ba)^3 = b\ ab\ ab\ a = 0$
$\rm(2)\quad c^2 = c\: \Rightarrow\: c\: $ central $ $ [lo que significa que tenemos $\rm\ \color{#C00}{xc = cx}\ $ para todos $\rm\:x$ ]
$\rm\begin{eqnarray}Proof:\quad c(x-cx) &=&\rm 0\:\Rightarrow\: (x-cx)c = 0\ \ by\ (1),\ \ so\ \ \color{#C00}{xc} = cxc\\ \rm (x-xc)c &=&\rm 0\:\Rightarrow\: c(x-xc) = 0\ \ by\ (1),\ \ so\ \ \color{#C00}{cx} = cxc\end{eqnarray}$
$\rm(3)\quad x^2\:$ central vía $\rm\:c = x^2\:$ en $(2)$
$\rm(4)\quad c^2 = 2c\:\Rightarrow\: c\:$ central. $\ $ Prueba: $\rm\:c = c^3 = 2c^2\:$ central por $(3)$ .
$\rm(5)\quad x+x^2\:$ central vía $\rm\:c = x + x^2\:$ en $(4)$
$\rm(6)\quad x = (x+x^2)-x^2\:$ central vía $(3),(5).\quad$ QED
Reposicionarla desde aquí . Tenga en cuenta que $R$ es no necesariamente unital.
Algunos datos generales:
Llamamos a un anillo $R$ , anillo en J ( Anillo Jacobson ), si para cualquier $x \in R$ hay un número natural $n(x) >1$ s.t. $x^{n(x)}=x$ . (De hecho, Jacobson ha demostrado que cualquier anillo J es conmutativo, para la prueba puede echar un vistazo a Anillos no conmutativos escrito por Herestein )
Lema 1 : Si $R$ sea un anillo J, entonces $N(R)= \{0 \}$ donde $N(R)$ denota el nilradical de $R$ .
Prueba : Dejemos que $0\not= x\in N(R)$ . Entonces hay un número natural más pequeño que $1$ s.t. $x^m=0$ . Desde $R$ es un anillo J, hay un $n>1$ s.t. $x^n=x$ . Dejemos que $m=nq+r$ donde $0 \leq r <n$ . Por lo tanto,
$$x^m=x^{nq+r}=(x^n)^qx^r=x^qx^r=x^{q+r}=0$$
Sin embargo, $q+r<m$ lo cual es una contradicción, ya que $m$ fue elegido para ser el número más pequeño que satisface $x^m=0$ .
Lema 2 : Supongamos que en un anillo $R$ , $N(R)= \{0 \}$ , entonces cualquier idempotente elemento $a$ es decir $a^2=a$ , se encuentra en el centro $Z(R)$ .
Prueba : Supongamos que $x \in R$ . Entonces
$$(axa-ax)^2=(axa-ax)(axa-ax)=axaaxa-axaxa-axaax+axax=axaxa-axaxa-axax+axax=0.$$
Desde $N(R)= \{0 \}$ entonces tenemos $axa-ax=0 \rightarrow axa=ax$ . Con el mismo enfoque y considerando $(axa-xa)^2$ obtendremos $axa=xa$ . Por lo tanto, $ax=xa$ y como $x$ era un elemento arbitrario de $R$ entonces $a \in Z(R)$ .
Lema 3 : En un anillo en J $R$ tenemos $x^{n(x)-1} \in Z(R).$
Prueba : $(x^{n(x)-1})^2=x^{2n(x)-2}=x^{n(x)}x^{n(x)-2}=xx^{n(x)-2}=x^{n(x)-1}$ . Así, $x^{n(x)-1}$ es un elemento idempotente de $R$ y por Lema 1 y 2. obtenemos el resultado.
En particular, en su pregunta, $n=n(x)=3$ y $x^2 \in Z(R),$ para cualquier $x \in R.$ Además
$$xy=(xy)^3=xyxyxy=x(yx)^2y=(yx)^2xy=yxyx^2y=yx^3y^2=yxy^2=y^3x=yx.$$
Ejercicio : La misma pregunta con $x^4=x$ para cualquier $x \in R.$
Esta no es la mejor prueba, pero permítanme añadirla porque aún no se ha mencionado. La prueba se basa en un solo identidad polinómica no conmutativa (cuya verificación puede hacerse rápidamente con un sistema de álgebra computacional).
Definir $f(x,y):=(x+y)^3-x^3-y^3 \in \mathbb{Z} \langle x,y \rangle$ . Entonces tenemos
$$f\bigl(x,y+(x \cdot y-y \cdot x)\bigr) - f(x,y) - f\bigl(x,(x \cdot y-y \cdot x)\bigr) - (x \cdot f(x,y) - f(x,y) \cdot x)$$ $$ = -x^3 \cdot y + y \cdot x^3$$ Así, si $R$ es un anillo con $a^3=a$ para todos $a \in R$ tenemos $f(a,b)=0$ para $a,b \in R$ y, por tanto, también $-a^3 \cdot b + b \cdot a^3=0$ resp. $ab=ba$ .
Resulta que he encontrado una serie de ejercicios recientes sobre muchos de los pequeños $n$ casos del Teorema de Jacobson. También ocurre que mi solución es diferente a las contenidas en el enlace de @lhf arriba.
Así que tenemos que $a^3 = a \quad\forall a \in R$ y así $2a = (a+a)^3 =8a$ Por lo tanto $6a = 0$ .
Ahora considere los ideales $2R$ y $3R$ . La intersección de $2R$ y $3R$ es trivial, como si $a \in 2R \cap 3R$ entonces $a = 2r = 3s$ para algunos $r,s$ . Así, $3a = 6r = 0 = 6s = 2a$ y así $(3-2)a = a= 0$ . Así que $2R \oplus 3R = R$ . Además, si $a \in 2R$ , $b \in 3R$ entonces $ab, ba \in 2R \cap 3R$ y así $ab = ba = 0$ . Así que sólo nos preocupamos de la conmutatividad en cada ideal por separado.
En $3R$ tenemos ambos $a^3 = a$ y $2a = 2 \cdot 3r = 0$ (algunos $r$ ). Entonces $1 + a = (1 + a)^3 = 1 + 3a + 3a^2 + a^3 = 1 + a + a^2 +a = 1 + a^2 \implies a^2 = a$ . ¿Y qué? En ese caso, también tenemos $(1 + a) = (1 + a)^2 = 1 + 2a + a^2 = 1 + 2a + a$ y así $2a = 0$ (sí, lo tenemos en nuestro ideal, pero esto es cierto en general en los anillos booleanos). Continuando, $(a + b) = (a + b)^2 = a^2 + ab + ba + b^2 = a + ab + ba + b$ Así que $ab = -ba = -ba + 2ba = ba$ .
Para $2R$ tenemos ambos $a^3 = a$ y $3a = 0$ . Entonces tenemos que $a + b = (a + b)^3 = a^3 + a^2b + aba + ab^2 + ba^2 + bab + b^2a + b^3 $$ = a + a^2b + aba + ab^2 + ba^2 + bab + b^2a + b $ on the one hand, and $ a - b = (a - b)^3 = a^3 - a^2b - aba + ab^2 - ba^2 + bab + b^2a - b^3 $ $ = a - a^2b - aba + ab^2 - ba^2 + bab + b^2a - b$.
Tomando la diferencia entre estos, vemos $2(a^2b + aba + ba^2) = 0$ y así $a^2 b + aba + ba^2 = 0$ . Multiplicar por $a$ y obtenemos $a^3b + a^2ba + aba^2 = ab + (a^2b + aba)a = ab + (-ba^2)a = ab - ba = 0$ . Así, $ab = ba$ .
Como ambos ideales conmutan por separado y en productos, $R$ los desplazamientos en general.
I-Ciencias es una comunidad de estudiantes y amantes de la ciencia en la que puedes resolver tus problemas y dudas.
Puedes consultar las preguntas de otros usuarios, hacer tus propias preguntas o resolver las de los demás.
11 votos
Si se trata de una tarea, ¿qué has probado? Si no, busca en Google "x3=x anillo conmutativo" y obtendrás varias soluciones, entre ellas mathematics.uni-bielefeld.de/~sillke/PUZZLES/herstein .
2 votos
También puede echar un vistazo a esto Pregunta de MathOverflow .
0 votos
Recuerdo que resolvimos este problema en clase como una divertida aplicación del teorema de la densidad de Jacobson. De alguna manera esto parece mejor (aunque un poco exagerado) que los cálculos ad hoc que dan las otras soluciones.
6 votos
Un ejercicio del libro de texto de Herstein Temas de Álgebra . Herstein dijo que, de todo el correo que recibió en relación con ese libro de texto, la gran mayoría era sobre este único ejercicio.
5 votos
mathoverflow.net/questions/29590/